LSAT and Law School Admissions Forum

Get expert LSAT preparation and law school admissions advice from PowerScore Test Preparation.

 Administrator
PowerScore Staff
  • PowerScore Staff
  • Posts: 8919
  • Joined: Feb 02, 2011
|
#40229
Complete Question Explanation
(The complete setup for this game can be found here: lsat/viewtopic.php?t=8561)

The correct answer choice is (E)

If the Trents owned exactly one of the buildings, then the 1-3-1 distribution applies. With Templates 2B and 3B in place, we can easily prove that F, M and S are the only buildings any one of which could be the building that the Trents owned. This prephrase agrees with answer choice (E), which is correct.

Without the use of templates, we can eliminate answer choice (C), because the Trents cannot have owned I in a 1-3-1 distribution (see inference above). We can also eliminate answer choices (A), (B), and (D), because the Trents can own S in a 1-3-1 distribution—as shown in the local solution to Question #17 (never erase your work!). Thus, answer choice (E) is the only remaining contender.

Answer choice (A) is incorrect, because it contains an incomplete list of the buildings, any one of which could be the only building owned by the Trents. As discussed above, the Trents could also have owned M (and no other building), as well as S (and no other building).

Answer choice (B) is incorrect, because it contains an incomplete list of the buildings, any one of which could be the only building owned by the Trents. As discussed above, the Trents could also have owned S (and no other building).

Answer choice (C) is incorrect, because it contains a list that is both inaccurate and incomplete. The list is inaccurate, because the Trents cannot have owned only I (that would directly violate the last rule). The list is also incomplete, because the Trents could also have owned M (and no other building).

Answer choice (D) is incorrect, because it contains a list that is both inaccurate and incomplete. The list is inaccurate, because the Trents cannot have owned only G in a 1-3-1 distribution (see Templates). The list is also incomplete, because the Trents could also have owned S (and no other building).

Answer choice (E) is the correct answer choice. See explanation above.

Get the most out of your LSAT Prep Plus subscription.

Analyze and track your performance with our Testing and Analytics Package.